Confusing (1 Viewer)

Rmd_1

Member
Joined
Mar 8, 2004
Messages
36
Location
sydney
it's not a hard question but i just can't work through it
Find the area between the curve y=ln x , the y-axis and the line
y=2 and y=4, correct to 3 significant figures..

answer is 47.2 ..
 

CM_Tutor

Moderator
Moderator
Joined
Mar 11, 2004
Messages
2,644
Gender
Male
HSC
N/A
The area in question is against the y-axis, so the required area is:

int (from 2 to 4) x dy, where y = ln x, and so x = e^y
= int (from 2 to 4) e^y dy
= [ e^y ] (from 2 to 4)
= e^4 - e^2
= 47.209 ... by calculator
= 47.2 (to 3 sig fig).
 

Rmd_1

Member
Joined
Mar 8, 2004
Messages
36
Location
sydney
one more questsion, how can i solve it if it asked to find out the volume of the solid which rotated about the y-axis, from y=1 to y=3, the curve is y=lnx
 

CM_Tutor

Moderator
Moderator
Joined
Mar 11, 2004
Messages
2,644
Gender
Male
HSC
N/A
Volume about the y-axis, so the volume is given by int (from a to b) pi * x^2 dy. Here, that is:

int (from 1 to 3) pi * x^2 dy, where y = ln x, so x = e^y
= pi * int (from 1 to 3) (e^y)^2 dy
= pi * int (from 1 to 3) e^2y dy
= pi * [e^2y / 2] (from 1 to 3)
= (pi / 2) * [e^2(3) - e^2(1)]
= (pi / 2) * (e^6 - e^2)
= pi * e^2 (e^4 - 1) / 2 cu units (in exact form)

This is approximately 622 cu units (3 sig fig).
 

Rmd_1

Member
Joined
Mar 8, 2004
Messages
36
Location
sydney
= pi * int (from 1 to 3) e^2y dy
= pi * [e^2y / 2] (from 1 to 3)

but how cau u get from e^2y dy to [e^2y / 2]
is there have any theorem or what? i learn it by myself.
 

Heinz

The Musical Fruit
Joined
Oct 6, 2003
Messages
419
Location
Canberra
Gender
Male
HSC
2004
the derivative of e^f(x) = f'(x)*e^f(x). Similarly, the integral of e^f(x) = e^f(x)/f'(x)
 

Rmd_1

Member
Joined
Mar 8, 2004
Messages
36
Location
sydney
Okey, i got it , it's a little complicate,isn't it

anyway, thank u CM_Tutor and Heinz !
 

CM_Tutor

Moderator
Moderator
Joined
Mar 11, 2004
Messages
2,644
Gender
Male
HSC
N/A
The integral of e^f(x) is not repeat NOT e^f(x) / f'(x). This is a shortuct that some people use, that happens to work for a variety of HSC questions - including the one that Rmd_1 is aking about. Nevertheless, IT SHOULD NOT BE USED, as it is not mathematically valid - I know some teachers teach it, but that doesn't make it true.

Breathing out, and calming down ...

Sorry Heinz, you've hit on one of my pet hates. :)
Originally posted by Rmd_1
= pi * int (from 1 to 3) e^2y dy
= pi * [e^2y / 2] (from 1 to 3)

but how cau u get from e^2y dy to [e^2y / 2]
is there have any theorem or what? i learn it by myself.
I have used the result that the integral of e^(Ax + B) with respect to x is e^(Ax + B) / A, when A and B are constants.

In the above, I am integrating with respect to y, so use y instead of x, with A = 2 and B = 0.

The three integral results you need to know for exponential functions (ignoring arbitrary constants) are:

the integral of e^x with respect to x is e^x

the integral of e^(Ax + B) with respect to x is e^(Ax + B) / A, where A and B are constants

the integral of f'(x) * e^f(x) with respect to x is e^f(x)
 

Rmd_1

Member
Joined
Mar 8, 2004
Messages
36
Location
sydney
one more question : )

By considering coefficient of x^4 on both sides of
(1+x)^20=(1+x)^10(1+x)^10 , show that
20C4=2(10C0*10C4+10C1*10C3)+(10C2)^2.

that's a little bit hard
 

Heinz

The Musical Fruit
Joined
Oct 6, 2003
Messages
419
Location
Canberra
Gender
Male
HSC
2004
Originally posted by CM_Tutor
The integral of e^f(x) is not repeat NOT e^f(x) / f'(x). This is a shortuct that some people use, that happens to work for a variety of HSC questions - including the one that Rmd_1 is aking about. Nevertheless, IT SHOULD NOT BE USED, as it is not mathematically valid - I know some teachers teach it, but that doesn't make it true.

Breathing out, and calming down ...

Sorry Heinz, you've hit on one of my pet hates. :)
I dont like it when teachers yell at me :( Oh and blame J.B. fitzpatrick, thats the text i used when i did 2u. *flicks through text* pg 409 :)
 

CM_Tutor

Moderator
Moderator
Joined
Mar 11, 2004
Messages
2,644
Gender
Male
HSC
N/A
I am not a teacher, and i am trying to help you (and others). The result you claim is a dangerous and (unfortunately) common misconception. It works for most HSC questions, and that just makes it more dangerous, as it allows it to spread. As I said, it is one of my pet hates.

PS: I just looked at p 409 of 2u Fitzpatrick, and I don't see this result claimed.
 

Heinz

The Musical Fruit
Joined
Oct 6, 2003
Messages
419
Location
Canberra
Gender
Male
HSC
2004
Originally posted by CM_Tutor
I am not a teacher, and i am trying to help you (and others). The result you claim is a dangerous and (unfortunately) common misconception. It works for most HSC questions, and that just makes it more dangerous, as it allows it to spread. As I said, it is one of my pet hates.

PS: I just looked at p 409 of 2u Fitzpatrick, and I don't see this result claimed.
well its sorta like that? lol. (im slowly digging my own grave arent i?)
 

Rmd_1

Member
Joined
Mar 8, 2004
Messages
36
Location
sydney
i'm sorry for that!
Ok, i'll get through bymyself.
thank u GM_Tutor. Just don't blame me,OK? :(
 

CM_Tutor

Moderator
Moderator
Joined
Mar 11, 2004
Messages
2,644
Gender
Male
HSC
N/A
The result you claim, that the integral of e^f(x) is e^f(x) / f'(x) is true ONLY if f(x) is a linear function in x, ie if
f(x) = Ax + B, for A and B constants. Fitzpatrick, p. 409, wrote that the integral of e^(kx) is (1 / k) * e^(kx) + c, which is of the same form as your result, but is not the same thing.

Your result is mostly used as a short cut for questions like integrate xe^(x^2). This is not a function of the form e^f(x), but people use your result anyway, and say the answer is xe^(x^2) / 2x + c, and then cancel the x's to get e^(x^2) / 2 + c, which is the correct answer, but the working is fundamentally flawed. Whether this gets penalised in an exam situation depends on the marker.

The correct way to do this type of question is using the third result I stated about, namely that the integral of
f'(x) * e^f(x) is e^f(x). This is done as follows: First, recognise that e^(x^2) is of the form e^f(x), with f(x) = x^2, so this result is probably needed. We need an f'(x), which is 2x, so start by rewriting the question. So ...

int xe^(x^2) dx = (1 / 2) * int 2x * e^(x^2) dx [Notice that we now have an f'(x) * e^f(x)]
= (1/2) * e^(x^2) + C, for some constant C [I integrated f'(x) * e^f(x) to e^f(x)]
= e^(x^2) / 2 + C, as expected.

Now, why is your result so dangerous? Well, it leads people to think things like int e^(x^2) dx = e^(x^2) / 2x, which is totally wrong - and that's just for starters.
 
Last edited:

Heinz

The Musical Fruit
Joined
Oct 6, 2003
Messages
419
Location
Canberra
Gender
Male
HSC
2004
Originally posted by Rmd_1
one more question : )

By considering coefficient of x^4 on both sides of
(1+x)^20=(1+x)^10(1+x)^10 , show that
20C4=2(10C0*10C4+10C1*10C3)+(10C2)^2.

that's a little bit hard

(1+x)^20 = (1+x)^10*(1+x)^10

LHS =(20C0*x^0 + 20C1*x + 20C@*x^2+20C3*x^3 + 20C4*x^4...) You wont need to worry about the rest of the expansion as its not used. Just go up to the required term.
:. the coefficient of x^4 is 20C4

RHS = (10C0*x^0 + 10C1*x + 10C2*x^2 + 10C3*x^3 +10C4*x^4...)*(10C0*x^0 + 10C1*x + 10C2*x^2 + 10C3*x^3 +10C4*x^4...)

C*x^4 = (10C0*x^0)*(10C4*x^4) + (10C1*x)*(10C3*x^3) + (10C2*x^2)*(10C2*x^2) + (10C3*x^3)*(10C1*x) + (10C4*x^4)*(10C0*x^0)

:. the coefficient of x^4 =2(10C0*10C4 + 10C1*10C3) + (10C2)^2
 
Last edited:

Users Who Are Viewing This Thread (Users: 0, Guests: 1)

Top